0 Daumen
653 Aufrufe

Aufgabe:

a)

\( \displaystyle\sum\limits_{k=1}^{\infty}{\dfrac{3-(-2)^{3k-2}}{3^{2k-1}}} \)

b)

\( \displaystyle\sum\limits_{k=1}^{\infty}{\dfrac{3^{2k-1}+(-4)^{2k+1}}{2^{4k+3}}} \)

c)

\( \displaystyle\sum\limits_{k=1}^{\infty}{(-1)^k}\, \dfrac{k-1}{k+1} \)

d) 

\( \displaystyle\sum\limits_{k=1}^{\infty}{\frac{1}{k^2+k}} \)


Problem/Ansatz:

Ich benötige euer Hilfe,da ich an dieser Aufgabe hänge. Ich habe keine Idee wie ich anfangen soll und das Skript hilft mir auch nicht weiter. Kann mir jemand erklären, wie ich vorgehen muss? Das wäre sehr nett

Avatar von

Vom Duplikat:

Titel: Welche der folgenden Reihen sind konvergent? Berechnen deren Summe!

Stichworte: reihen,analysis,konvergent,exponenten,grenzwert

Aufgabe:

a)

\( \sum\limits_{k=1}^{\infty}{\frac{3-(-2)^{3k-2}}{3^{2k-1}}} \)

b)

\( \sum\limits_{k=1}^{\infty}{\frac{3^{2k-1}+(-4)^{2k+1}}{2^{4k+3}}} \)

c)

\( \sum\limits_{k=1}^{\infty}{(-1)} \)k \( \frac{k-1}{k+1} \)

d) 

\( \sum\limits_{k=1}^{\infty}{\frac{1}{k²+k}} \)


Problem/Ansatz:

leider hänge ich an dieser Aufgabe und komme nicht voran. Daher meine Frage, ob mir denn jemand einen Rechenweg vorschlagen könnte? Da mir die Skripte auch nicht viel weiterhelfen.

Vielen Dank an Euch

Das Nullfolgenkriterium wäre ein Anfang.

1 Antwort

+1 Daumen
 
Beste Antwort

\( \displaystyle\sum\limits_{k=1}^{\infty}{\dfrac{3-(-2)^{3k-2}}{3^{2k-1}}} \)

\( \displaystyle=\sum\limits_{k=1}^{\infty}{\dfrac{3}{3^{2k-1}}} -\sum\limits_{k=1}^{\infty}{\dfrac{(-2)^{3k-2}}{3^{2k-1}}} \)

\( \displaystyle=\sum\limits_{k=1}^{\infty}{\dfrac{1}{3^{2k-2}}} -\dfrac{3}{4}\sum\limits_{k=1}^{\infty}{\dfrac{(-2)^{3k}}{3^{2k}}} \)

\( \displaystyle=\sum\limits_{k=1}^{\infty}{\dfrac{1}{9^{k-1}}} -\dfrac{3}{4}\sum\limits_{k=1}^{\infty}{(\dfrac{-8}{9})^k} \)

\( \displaystyle=\sum\limits_{k=0}^{\infty}{\dfrac{1}{9^{k}}} -\dfrac{3}{4}(-1+\sum\limits_{k=0}^{\infty}{(\dfrac{-8}{9})^k)} \)

Jetzt hast du zwei geometrische Reihen, einmal mit q=1/9 und einmal mit q=(-8/9)

Die geben 9/8  bzw  9/17. Also :

\( \displaystyle= \dfrac{9}{8}-\dfrac{3}{4}(-1+\dfrac{9}{17}) =\dfrac{201}{136}\)

Die 2. geht sicher ähnlich. Allerdings divergent !

c) s. Kommentar.

d) Beachte   1 / (k^2 + k) = 1/k  -   1/(k+1) , also Teleskopsumme.

Ergebnis also 1.

Avatar von 288 k 🚀

Ein anderes Problem?

Stell deine Frage

Willkommen bei der Mathelounge! Stell deine Frage einfach und kostenlos

x
Made by a lovely community